\lecture{11}{2023-11-23}{Cardinals} \subsection{Cardinals} \begin{definition} Let $a$ be any set. The \vocab{cardinality} of $a$ denoted by $\overline{\overline{a}}$, $|a|$ or $\card(a)$, is the smallest ordinal $\alpha$ such that there is some bijection $f\colon \alpha \to a$. An ordinal $\alpha$ is called a \vocab{cardinal}, iff \gist{% there is some set $a$ with $|a| = \alpha$ (equivalently, $|\alpha| = \alpha$).% }{$|\alpha| = \alpha$.} \end{definition} We often write $\kappa, \lambda, \ldots$ for cardinals. \begin{lemma} For every cardinal $\kappa$, there is come cardinal $\lambda > \kappa$. \end{lemma} \begin{proof} \gist{% Consider the powerset of $\kappa$. We know that there is no surjection $\kappa \twoheadrightarrow \cP(\kappa)$. Hence $\kappa < |2^{\kappa}|$. }{$\kappa < |2^{\kappa}|$.} \end{proof} \begin{definition} For each cardinal $\kappa$, $\kappa^+$ denotes the least cardinal $\lambda > \kappa$. \end{definition} \gist{% \begin{warning} This has nothing to do with the ordinal successor of $\kappa$. \end{warning} }{} \begin{lemma} Let $X$ be any set of cardinals. Then $\sup X$ is a cardinal. \end{lemma} \begin{proof} \gist{% If there is some $\kappa \in X$ with $\lambda \le \kappa$ for all $\lambda \in X$, then $\kappa = \sup(X)$ is a cardinal. Let us now assume that for all $\kappa \in X$ there is some $\lambda \in X$ with $\lambda > \kappa$. Suppose that $\sup(X)$ is not a cardinal and write $\mu = |\sup(X)|$. Then $\mu \in \sup(X)$, since $\sup(X)$ is an ordinal. However $\sup(X)$ is the least ordinal larger than all $\alpha \in X$, so there is $\lambda \in X$ with $\lambda > \mu$. However, there exists $\mu \twoheadrightarrow \sup(X)$, hence also $\mu \twoheadrightarrow \lambda$ (which is in contradiction to $\lambda$ being a cardinal). }{% \begin{itemize} \item If $\sup(X) \in X$ this is trivial. \item Let $\sup(X) \not\in X$, $\mu \coloneqq |\sup(X)|$. \begin{itemize} \item Suppose that $\sup(X)$ is not a cardinal. Then $\mu \in \sup(X)$, since $\sup(X) \in \OR$. \item $\exists \lambda \in X.~\lambda > \mu$ $\lightning$. \end{itemize} \end{itemize} } \end{proof} We may now use the \yaref{thm:recursion} to define a sequence $\langle \aleph_\alpha : \alpha \in \OR \rangle$ with the following properties: \begin{IEEEeqnarray*}{rCl} \aleph_0 &=& \omega,\\ \aleph_{\alpha+1} &=& (\aleph_\alpha)^+,\\ \aleph_{\lambda} &=& \sup \{\aleph_\alpha : \alpha < \lambda\}. \end{IEEEeqnarray*} Each $\aleph_\alpha$ is a cardinal. Also, a trivial induction shows that $\alpha \le \aleph_\alpha$. In particular $|\alpha| \le \aleph_{\alpha}$. Therefore the $\aleph_\alpha$ are all the infinite cardinals: If $a$ is any infinite set, then $|a| \le \aleph_{|a|}$, so $|a| = \aleph_\beta$ for some $\beta \le |\alpha|$. \begin{notation} Sometimes we write $\omega_\alpha$ for $\aleph_\alpha$ (when viewing it as an ordinal). \end{notation} \begin{notation} Let $\leftindex^a b \coloneqq \{f : f \text{ is a function}, \dom(f) = a, \ran(f) \subseteq b\}$. \end{notation} \begin{definition}[Cardinal arithmetic] Let $\kappa$, $\lambda$ be cardinals. Define \begin{IEEEeqnarray*}{rCl} \kappa + \lambda &\coloneqq & |\{0\} \times \kappa \cup \{1\} \times \lambda|,\\ \kappa \cdot \lambda &\coloneqq & |\kappa \times \lambda|,\\ \kappa^{\lambda} &\coloneqq & |\leftindex^{\lambda}\kappa|. \end{IEEEeqnarray*} \end{definition} \gist{% \begin{warning} This is very different from ordinal arithmetic! \end{warning} }{} \begin{theorem}[Hessenberg] \label{thm:hessenberg} For all $\alpha$ we have \[ \aleph_\alpha \cdot \aleph_\alpha = \aleph_\alpha. \] \end{theorem} \begin{corollary} For all $\alpha, \beta$ it is \[ \aleph_\alpha + \aleph_\beta = \aleph_\alpha \cdot \aleph_\beta = \max \{\aleph_\alpha, \aleph_\beta\}. \] \end{corollary} \begin{proof} Wlog.~$\alpha \le \beta$. \gist{% Trivially $\aleph_\alpha \le \aleph_\beta$. It is also clear that \[ \aleph_{\beta} \le \aleph_\alpha + \aleph_{\beta} \le \aleph_\alpha \cdot \aleph_\beta \le \aleph_\beta \cdot \aleph_\beta = \aleph_\beta. \] }{Then $\aleph_{\beta} \le \aleph_\alpha + \aleph_{\beta} \le \aleph_\alpha \cdot \aleph_\beta \le \aleph_\beta \cdot \aleph_\beta = \aleph_\beta$.} \end{proof} \begin{refproof}{thm:hessenberg} \gist{% Define a well-order $<^\ast$ on $\OR \times \OR$ by setting \[ (\alpha,\beta) <^\ast (\gamma,\delta) \] iff \begin{itemize} \item $\max(\alpha,\beta) < \max(\gamma,\delta)$ or \item $\max(\alpha,\beta) = \max(\gamma,\delta)$ and $\alpha < \gamma$ or \item $\max(\alpha,\beta) = \max(\gamma,\delta)$ and $\alpha = \gamma$ and $\beta < \delta$. \end{itemize} It is clear that this is a well-order. There is an isomorphism \[ (\OR, <) \cong^{\Gamma^{-1}} (\OR \times \OR, <^\ast). \] $\Gamma$ is called the \vocab{Gödel pairing function}. \begin{claim} For all $\alpha$ it is $\ran(\Gamma\defon{\aleph_\alpha \times \aleph_\alpha}) = \aleph_\alpha$, i.e. \[ \aleph_\alpha = \{\xi: \exists \eta, \eta' < \aleph_\alpha.~\xi = \Gamma((\eta,\eta'))\}. \] \end{claim} \begin{subproof} We use induction of $\alpha$. The claim is trivial for $\alpha = 0$. Now let $\alpha > 0$ and suppose the claim to be true for all $\beta < \alpha$. It is easy to see that \[ \ran(\Gamma\defon{\aleph_\alpha \times \aleph_\alpha}) \supseteq \aleph_\alpha, \] as otherwise $\Gamma\defon{\aleph_\alpha \times \aleph_\alpha}: \aleph_{\alpha} \times \aleph_\alpha \to \eta$ would be a bijection for some $\eta < \aleph_\alpha$, but $\aleph_\alpha$ is a cardinal. Suppose that $\ran(\Gamma\defon{\aleph_\alpha \times \aleph_\alpha}) \supsetneq \aleph_\alpha$. Then there exist $\eta, \eta' < \aleph_\alpha$ with \[ \Gamma((\eta, \eta')) = \aleph_\alpha. \] So $\Gamma\defon{\{(\gamma,\delta) : (\gamma,\delta) <^\ast (\eta, \eta'\}}$ is bijective onto $\aleph_\alpha$. If $(\gamma,\delta) <^\ast (\eta, \eta')$, then $\max \{\gamma,\delta\} \le \max \{\eta, \eta'\}$. Say $\eta \le \eta' < \aleph_\alpha$ and let $\aleph_\beta = |\eta'|$. There is a surjection \[f\colon \underbrace{(\eta +1)}_{ \le \aleph_\beta} \times \underbrace{(\eta' + 1)}_{\sim \aleph_\beta} \twoheadrightarrow \aleph_\alpha.\] This gives rise to a surjection $f^\ast \colon \aleph_\beta \times \aleph_\beta \to \aleph_\alpha$. The inductive hypothesis then produces a surjection $f^\ast\colon \aleph_\beta \to \aleph_\alpha \lightning$. \end{subproof} }{ \begin{itemize} \item Define wellorder $<^\ast \subseteq \OR \times \OR$, $(\alpha,\beta) <^\ast (\gamma,\delta)$ iff \begin{itemize} \item $\max(\alpha,\beta) < \max(\gamma,\delta)$ or \item $\max(\alpha,\beta) = \max(\gamma,\delta)$ and $\alpha < \gamma$ or \item $\max(\alpha,\beta) = \max(\gamma,\delta)$ and $\alpha = \gamma$ and $\beta < \delta$. \end{itemize} \item Gödel pairing function $\Gamma : (\OR \times \OR, <^\ast) \xrightarrow{\cong} (\OR, <)$. \item $\forall \alpha.~\ran(\Gamma\defon{\aleph_\alpha \times \aleph_\alpha}) = \aleph_\alpha$. \begin{itemize} \item Induction on $\alpha$. \item Clearly $\aleph_\alpha \subseteq \ran(\Gamma\defon{\aleph_\alpha \times \aleph_\alpha})$ ($\aleph_\alpha$ is a cardinal). \item Suppose $\aleph_\alpha \subsetneq \ran(\Gamma\defon{\aleph_\alpha \times \aleph_\alpha})$, We get a surjection $\aleph_\beta \xrightarrow{\text{induction}} \aleph_\beta \times \aleph_\beta \to \aleph_\alpha$ for some $\beta < \alpha$. \end{itemize} \end{itemize} } \end{refproof} \gist{% However, exponentiation of cardinals is far from trivial: \begin{observe} $2^{\kappa} = |\cP(\kappa)|$, since $\leftindex^{\kappa} \{0,1\} \leftrightarrow\cP(\kappa)$. Hence by Cantor $2^{\kappa}\ge \kappa^+$. \end{observe} This is basically all we can say. }{} The \vocab{continuum hypothesis} states that $2^{\aleph_0} = \aleph_1$.